Wie erhält man die explizite Form der Greenschen Funktion der Klein-Gordon-Gleichung?

Die Definition der Green-Funktion für die Klein-Gordon-Gleichung lautet:

( T 2 2 + M 2 ) G ( X , T ) = δ ( T ) δ ( X )
Nach diesen Ressourcen:

  1. Green-Funktion für die inhomogene Klein-Gordon-Gleichung , die Green-Funktion sieht folgendermaßen aus:

G ( X , T ) = θ ( T ) 2 π δ ( T 2 | X | 2 ) M 2 π θ ( T | X | ) J 1 ( M T 2 | X | 2 ) M T 2 | X | 2

  1. Aus der Wikipedia gibt es mehrere Arten von Propagatoren , die von der Wahl der Kontur abhängen. Sie sehen dem obigen sehr ähnlich, sind aber immer noch nicht identisch mit mir. Zum Beispiel sieht die Funktion des fortgeschrittenen und verzögerten Grüns dem obigen Ausdruck sehr ähnlich.

Ich möchte die explizite Form der oben aufgeführten grünen Funktion ableiten , hier ist, was ich versucht habe:

Annehmen:

G ( X , T ) =   D 3 P   D ω e ich ( P X ω T ) G ( P , ω )

Als Ersatz für die Klein-Gordon-Gleichung erhält man:

G ( P , ω ) = 1 ( 2 π ) 4 1 ω 2 P 2 M 2

Dann:

G ( X , T ) = 1 ( 2 π ) 4   D 3 P   D ω e ich ( P X ω T ) 1 ω 2 P 2 M 2

In der obigen Formel befindet sich der Pol auf der realen Achse. Um eine endliche Antwort zu erhalten, muss man den Pol leicht von der realen Achse weg manipulieren. Bei Feynmans Wahl hat man den linken Pol leicht oben und den rechten leicht unten, wie folgt:Geben Sie hier die Bildbeschreibung ein

Dann haben wir:

G ( X , T ) = 1 ( 2 π ) 4   D 3 P   D ω e ich ( P X ω T ) 1 ω 2 P 2 M 2 + ich ϵ über integrieren  ω = θ ( T ) ich ( 2 π ) 3   D 3 P e ich P X e ich P 2 + M 2 T 2 P 2 + M 2 + θ ( T ) ich ( 2 π ) 3   D 3 P e ich P X e ich P 2 + M 2 T 2 P 2 + M 2 über integrieren  ϕ  Und  θ = ich ( 2 π ) 2 θ ( T ) | X | 0 P Sünde ( P | X | ) e ich P 2 + M 2 T P 2 + M 2 + ich ( 2 π ) 2 θ ( T ) | X | 0 P Sünde ( P | X | ) e ich P 2 + M 2 T P 2 + M 2

Die obige Ableitung scheint keinen Fehler zu haben und ich weiß nicht, wie ich vorgehen soll P integral, und ich kann die Ähnlichkeit der aktuellen Formel mit der geschlossenen Formel in den Ressourcen 1 und 2 nicht erkennen.

Integrieren P , fand ich, dass dieses Integral aus dem Buch hilfreich sein könnte:

0 e β γ 2 + X 2 cos B X = β γ γ 2 + β 2 K 1 ( γ β 2 + B 2 ) ,  mit:  R e β > 0 , R e γ > 0

Bearbeiten2

---um die aktualisierte Antwort von @Solenodon Paradoxus anzusprechen

Die Antwort schlug vor, die Kontur gegen den Uhrzeigersinn zu drehen ( ω = ich ω ' ), Deshalb:

G F ( P , ω ) = 1 ω ' 2 | P | 2 M 2 + ich ϵ = 0 D L e L ( ω ' 2 + | P | 2 + M 2 ich ϵ )

Stecken Sie die oben G F ( P , ω ) in das Integral von vier Impulsen:

G F ( X , T ) = ich ( 2 π ) 4 0   D L e ( M 2 ich ϵ ) L D ω ' D 3 P e L ω ' 2 + T ω ' e L P 2 + ich X P = ich 16 π 2 0   D L e ( M 2 ich ϵ ) L 1 L 2 e τ 2 4 L  mit  τ 2 = T 2 | X | 2

Seit jetzt ϵ Es ist unwichtig, das obige Integral zu führen, lassen Sie es einfach weg.

G F ( X , T ) = ich 16 π 2 0 D L e M 2 L 1 L 2 e τ 2 4 L = ich M 2 16 π 2 0 D ξ e ξ e M 2 τ 2 4 ξ 1 ξ 2
Wenn τ 2 > 0 , der Integrand divergiert bei ξ 0 , kann das Integral nicht durchgeführt werden.

Wenn τ 2 < 0 , wir haben:

ich M 4 π 2 τ 2 K 1 ( M τ 2 )
Wo K 1 modifizierte Bessel-Funktion ist, ist dies genau die Form des Feynmann-Propagators, wenn τ 2 < 0 .

Frage: Obwohl unser Ergebnis auf der einen Seite triumphiert, was ist mit der anderen Seite ( τ 2 > 0 )? Wie können wir es aus dem obigen Verfahren erhalten? In welchem ​​Schritt haben wir diese Möglichkeit ausgeschlossen?

Antworten (2)

Die Funktionen von Green sind nicht eindeutig. Jede Lösung davon erfüllt die homogene Gleichung,

( T 2 2 + M 2 ) F = 0
im interessierenden Bereich kann zur Green-Funktion addiert werden, ohne die inhomogene Gleichung zu zerstören. Der homogene Anteil muss also durch Randbedingungen festgelegt werden. Sie können den Bessel-Funktionsteil der Feynman-Version der Green-Funktion erhalten, indem Sie für die stationäre Green-Funktion mit einem 4-dimensionalen Raum auflösen, das heißt:
( 1 R 3 R [ R 3 R ] + M 2 ) G ( R ) = ich Ω 4 R 3 δ ( R )
und analytisch weiter zur imaginären Zeit. Innerhalb des vorderen Lichtkegels, wo das Argument von K imaginär ist, ergibt eine Hankel-Funktion erster Art:
M 4 π 2 τ 2 K 1 ( ich M τ 2 ) = M 8 π τ 2 H 1 ( 2 ) ( M τ 2 ) = M 8 π τ 2 [ J 1 ( M τ 2 ) ich Y 1 ( M τ 2 ) ] ,
mit J A Und Y A die Bessel-Funktionen erster bzw. zweiter Art.

Da sowohl der Operator als auch der inhomogene Teil reell sind, muss der Imaginärteil der Greenschen Funktion eine Lösung der homogenen Gleichung sein und der Realteil muss die Inhomogenität lösen. Das ist:

( T 2 2 + M 2 ) Ich bin { M 4 π 2 τ 2 K 1 ( ich M τ 2 ) } = 0 ,   A N D ( T 2 2 + M 2 ) Betreff { M 4 π 2 τ 2 K 1 ( ich M τ 2 ) } = δ 4 ( X ) ,
Die Funktion von Green ist also der Realteil dieses Ausdrucks.

Merkmale, die an dieser Stelle in diesem Beitrag noch gezeigt werden müssen: dass die Aufteilung in retardierte und fortgeschrittene Propagatoren gültig ist (erforderlich, um die Kausalität zu wahren) und die Nullmassengrenze des Propagators die korrekte Grenze ergibt. Die durch den Impuls injizierte Energie kann nicht verfolgt werden, da sie unendlich ist.

Vielleicht sollte die zweite Gleichung geschrieben werden als:
[ 1 R 3 R R 3 R + M 2 ] G ( R ) = δ ( R )
?
Guter Fang bei den falsch gesetzten Klammern. Die rechte Seite benötigte jedoch nur eine Vorzeichenkorrektur. Es ist eine 4-dimensionale Delta-Funktion, also muss sie den 4-dimensionalen Raumwinkel und den Faktor von haben R 3 .
Ich denke immer noch, dass sich die rechte Seite ändern sollte, die ursprüngliche Gleichung lautet:
( T 2 2 + M 2 ) G ( X , T ) = δ ( T ) δ ( X )
Sie möchten eine Vertretung vornehmen T ich w , wird die obige Gleichung zu:
( 4 2 + M 2 ) G ( R ) = δ ( ich w ) δ ( X )
Jetzt 4 2 = 1 R 3 R ( R 3 R ) + L 2 R 2 , Wo L 2 ist der Operator, der den Winkelteil differenziert. Wenn wir davon ausgehen G ( R ) = G ( R ) , L 2 darauf zu reagieren gibt nur null. Daher wird die Gleichung:
( 1 R 3 R ( R 3 R ) + M 2 ) G ( R ) = δ ( ich w ) δ ( X )
Eine Frage , in diesem Stadium, denke ich, dass wir darüber nachdenken w eine reelle Variable sein. Wenn es jedoch darum geht δ ( ich w ) (gut definiert?), haben wir die Identität that δ ( ich w ) = ich δ ( w ) ? Daher sollte die rechte Seite wahrscheinlich als geändert werden ich δ ( R ) ?
Tatsächlich habe ich das vermisst ich beim Übergang zum euklidischen 4-dimensionalen Raum. Und ja, das haben wir δ ( ich w ) = ich δ ( w ) .
Was ist mit dem ersten Kommentar, denke ich 1 R 3 Ω 4 ist unnötig.
Ah, dieser Teil ist falsch. Eine Deltafunktion muss der Identität gehorchen:
δ ( X ) D v = 1 ,
unabhängig von der Dimensionalität von X . Das heißt, wenn Sie Variablen ändern, zum Beispiel in ein sphärisches Koordinatensystem, muss die Delta-Funktion durch die Determinante der Jacobi-Zahl dividiert werden , um den mit dem Volumenelement verbundenen Faktor aufzuheben. In diesem Fall, D v = Ω 4 R 3 D R .
Ich habe einen vollständigen Beweis als Q&A gepostet: physical.stackexchange.com/questions/280212/… .

Sie könnten versuchen, die richtige Zeitmethode zu verwenden:

1 P 2 M 2 + ich ε = 0 D L exp [ L ( P 2 M 2 + ich ε ) ] .

Der Trick besteht darin, zuerst die (Gaußschen) Impulsintegrale zu erstellen und dann mit dem Integral fortzufahren L . Dies sollte die Bessel-Funktion ergeben. Lass es mich in den Kommentaren wissen, wenn du weitere Fragen hast.

UPDATE: Wie geht man mit der Lorentz-Signatur im Propagator um?

Wir interessieren uns für folgendes Integral:

D ( X ) = D 4 P ( 2 π ) 4 e ich P μ X μ P 2 M 2 + ich ε = D ω 2 π D 3 P ( 2 π ) 3 e ich ω T e ich P X ω 2 P 2 M 2 + ich ε .

Nun können wir die Integrationskontur in der komplexen Ebene so drehen, dass sie von keinem Pol durchquert wird. Ich werde die Kontur von Ihrem ursprünglichen Beitrag um 90 Grad gegen den Uhrzeigersinn drehen:

D ( X ) = ich D ω ' 2 π D 3 P ( 2 π ) 3 e ω ' T e ich P X ω ' 2 + P 2 + M 2 .

Ich habe auch an die neue Integrationsvariable übergeben: ω = ich ω ' . Jetzt ist der Nenner immer positiv und Sie können die richtige Zeitmethode verwenden. Alle Integrale sind Gaußsche und konvergente.

UPDATE 2: Die ich ε ermöglicht es uns, die Kontur gegen den Uhrzeigersinn zu drehen, verbietet jedoch andere (ähnliche) Transformationen, z. B. das Drehen im Uhrzeigersinn, wie Sie es getan haben. Denn bei seiner Verformung muss kein Pol die Kontur passieren. Somit, ω = ich ω ' , nicht ich ω ' . Glatte Transformationen der Kontur ändern das Integral nicht, solange kein Pol es passiert.

Also, die Rolle gespielt von ich ε ist zu bestimmen, wie das Integral in das euklidische Integral umgewandelt wird. Im euklidischen Fall (nachdem Sie die Kontur gedreht haben) können Sie das weglassen ich ε .

UPDATE 3: Da wir die Kontur gedreht haben, ω wird imaginär und ω ' = ich ω reell ist (der Sinn der Einführung von ω′ eigentlich). Somit, ω ' 2 + P 2 + M 2 kann nicht kleiner als null sein!

UPDATE 4: In Bezug auf Ihre endgültige Antwort vermute ich, dass Sie dies tun ω = ich ω ' und nicht ich ω ' (Du darfst nicht durch die Stange gehen, erinnerst du dich?), dann bekommst du die richtige Antwort. Ich bin mir natürlich nicht sicher, aber das würde ich tun.

Sie erhalten verschiedene Green-Funktionen. Es hängt wirklich alles davon ab, wie die Konturen um die Pole laufen. Oder wo man die einsteckt ich ε , wenn Sie wünschen.

UPDATE 5: Wie man damit umgeht τ 2 > 0 Fall?

Nun, hier ist, was ich mir ausgedacht habe. Sie könnten die Konturen der drei Integrale umdrehen P statt der über ω . Nach ähnlichen Berechnungen gelangen Sie zur zweiten Hankel-Funktion H 1 ( 2 ) . Alternativ könnten wir, da wir überall komplexe Zahlen verwenden, einfach die analytische Fortsetzung des Ergebnisses für machen τ 2 < 0 , was uns genau die gleiche Hankel-Funktion geben würde.

Ich kämpfe immer noch darum, eine klare Erklärung dafür zu finden, warum ein zusätzliches Delta-Symbol erscheint (es ist nicht so, als könnte es nicht erscheinen, da wir nur unseren Propagator für berechnet haben τ 2 > 0 Und τ 2 < 0 inzwischen).

Hast du das vermutet P 2 M 2 > 0 ? Wenn Sie wie bei Ihnen schreiben, werden beim Integral über den Impuls die Koeffizienten vor dem P 2 sollte im Realteil mindestens negativ sein.
seit der ω 2 P 2 Immer fortbestehen, die Integration der vier Impulse scheint mit dem Gaußschen Integral unmöglich zu sein.
@buzhidao in meiner Notation, P 2 = P 0 2 P 2 . NEIN, P 2 M 2 ist nicht immer größer als Null. Sie können die analytische Fortsetzung des Gaußschen Integrals verwenden.
Siehe meinen ersten Kommentar, ich denke, das Gaußsche Integral kann nur dann richtig definiert werden, wenn der Realteil des Koeffizienten des Quadrats kleiner als Null ist. Andersherum kann man das einfach nicht machen, es ist divergent.
@buzhidao Wenn Sie den euklidischen Fall betrachten, ist der (euklidische) Propagator proportional zu ( P 2 + M 2 ) 1 und alle Gaußschen Integrale sind wohldefiniert. Dann können Sie den Lorentz-Propagator durch die analytische Fortsetzung (Übergang zu den imaginären Zeitintervallen) erhalten, so wie Sie es in Ihrer Frage erklärt haben. Alternativ können Sie sich vorstellen, dass die Gaußschen Integrale auch dann durch analytische Fortsetzung definiert werden, wenn der Koeffizient vor dem quadratischen Term positiv ist.
@buzhidao Ich habe meine Antwort mit Details zum korrekten Umgang mit der Lorentz-Signatur aktualisiert. Bitte werfen Sie einen Blick darauf. Da dies eine Hausaufgaben- und Übungsaufgabe ist, werde ich nicht die vollständige Berechnung präsentieren. Die Details der Berechnung müssen Sie selbst vornehmen, sorry. Wenn Sie weitere Fragen haben, lassen Sie es mich bitte wissen.
Ich füge die Ableitung hinzu und listete meine Bedenken auf, bitte werfen Sie einen Blick darauf.
@buzhidao Ich habe meine Antwort noch einmal aktualisiert, bitte schau mal.
Ihre Erklärung ist ganz klar, ich habe die Konturdeformation als Substitution von Variablen missverstanden, führt daher zu der Sorge um das Divergenzproblem über die Eigenzeitmethode. Jetzt habe ich die Berechnung aktualisiert und ein neues Anliegen geäußert, bitte schauen Sie es sich an.
@buzhidao Ich muss darüber nachdenken. Sie werden sich innerhalb von 1-2 Tagen bei Ihnen melden.
@buzhidao Während ich darüber nachdenke, werfen Sie bitte einen Blick auf mein Update Nr. 5.